Solving functional equation $f(x)f(y) = f(x+y)$












4












$begingroup$


I'm having some trouble solving the following equation for $f: A rightarrow B$ where $A subseteq mathbb{R}$ and $B subseteq
mathbb{C}$ such as:



$$f(x)f(y) = f(x+y) quad forall x,y in A$$



The solution is supposed to be the form $f(x) = e^{kx}$ where k can be complex but I don't see how to show it.



Note: I'm trying to solve this equation to find irreductible representation of U(1) but it doesn't really matter I think










share|cite|improve this question











$endgroup$












  • $begingroup$
    Which set is $A$?
    $endgroup$
    – ajotatxe
    Jan 29 '15 at 21:58










  • $begingroup$
    For my case, it's $U(1) = lbrace e^{ivarphi} mid varphi in left[ 0, 2pi right] rbrace$ which can be represented by $ varphi in left[ 0, 2pi right]$ so I guess that A is $left[ 0, 2pi right]$
    $endgroup$
    – Samuel
    Jan 29 '15 at 22:02








  • 1




    $begingroup$
    @Samuel I would disagree with you on that, $U(1)$ is the circle, which is certainly different from $[0,2pi]$. In particular, $0$ and $2pi$ have the same image under $varphimapsto e^{ivarphi}$.
    $endgroup$
    – Hayden
    Jan 29 '15 at 22:04










  • $begingroup$
    So $left[ 0, 2pi right[$ ?
    $endgroup$
    – Samuel
    Jan 29 '15 at 22:05






  • 2




    $begingroup$
    Put $f(x) = e^{g(x)}$ then the equation becomes $g(x) + g(y) = g(x+y)$. The solution to this last equation can be found here (which shows that the only continuous solution is $g(x)=kx$)
    $endgroup$
    – Winther
    Jan 29 '15 at 22:09


















4












$begingroup$


I'm having some trouble solving the following equation for $f: A rightarrow B$ where $A subseteq mathbb{R}$ and $B subseteq
mathbb{C}$ such as:



$$f(x)f(y) = f(x+y) quad forall x,y in A$$



The solution is supposed to be the form $f(x) = e^{kx}$ where k can be complex but I don't see how to show it.



Note: I'm trying to solve this equation to find irreductible representation of U(1) but it doesn't really matter I think










share|cite|improve this question











$endgroup$












  • $begingroup$
    Which set is $A$?
    $endgroup$
    – ajotatxe
    Jan 29 '15 at 21:58










  • $begingroup$
    For my case, it's $U(1) = lbrace e^{ivarphi} mid varphi in left[ 0, 2pi right] rbrace$ which can be represented by $ varphi in left[ 0, 2pi right]$ so I guess that A is $left[ 0, 2pi right]$
    $endgroup$
    – Samuel
    Jan 29 '15 at 22:02








  • 1




    $begingroup$
    @Samuel I would disagree with you on that, $U(1)$ is the circle, which is certainly different from $[0,2pi]$. In particular, $0$ and $2pi$ have the same image under $varphimapsto e^{ivarphi}$.
    $endgroup$
    – Hayden
    Jan 29 '15 at 22:04










  • $begingroup$
    So $left[ 0, 2pi right[$ ?
    $endgroup$
    – Samuel
    Jan 29 '15 at 22:05






  • 2




    $begingroup$
    Put $f(x) = e^{g(x)}$ then the equation becomes $g(x) + g(y) = g(x+y)$. The solution to this last equation can be found here (which shows that the only continuous solution is $g(x)=kx$)
    $endgroup$
    – Winther
    Jan 29 '15 at 22:09
















4












4








4


3



$begingroup$


I'm having some trouble solving the following equation for $f: A rightarrow B$ where $A subseteq mathbb{R}$ and $B subseteq
mathbb{C}$ such as:



$$f(x)f(y) = f(x+y) quad forall x,y in A$$



The solution is supposed to be the form $f(x) = e^{kx}$ where k can be complex but I don't see how to show it.



Note: I'm trying to solve this equation to find irreductible representation of U(1) but it doesn't really matter I think










share|cite|improve this question











$endgroup$




I'm having some trouble solving the following equation for $f: A rightarrow B$ where $A subseteq mathbb{R}$ and $B subseteq
mathbb{C}$ such as:



$$f(x)f(y) = f(x+y) quad forall x,y in A$$



The solution is supposed to be the form $f(x) = e^{kx}$ where k can be complex but I don't see how to show it.



Note: I'm trying to solve this equation to find irreductible representation of U(1) but it doesn't really matter I think







calculus functional-equations






share|cite|improve this question















share|cite|improve this question













share|cite|improve this question




share|cite|improve this question








edited Jan 29 '15 at 23:10







Samuel

















asked Jan 29 '15 at 21:55









SamuelSamuel

1569




1569












  • $begingroup$
    Which set is $A$?
    $endgroup$
    – ajotatxe
    Jan 29 '15 at 21:58










  • $begingroup$
    For my case, it's $U(1) = lbrace e^{ivarphi} mid varphi in left[ 0, 2pi right] rbrace$ which can be represented by $ varphi in left[ 0, 2pi right]$ so I guess that A is $left[ 0, 2pi right]$
    $endgroup$
    – Samuel
    Jan 29 '15 at 22:02








  • 1




    $begingroup$
    @Samuel I would disagree with you on that, $U(1)$ is the circle, which is certainly different from $[0,2pi]$. In particular, $0$ and $2pi$ have the same image under $varphimapsto e^{ivarphi}$.
    $endgroup$
    – Hayden
    Jan 29 '15 at 22:04










  • $begingroup$
    So $left[ 0, 2pi right[$ ?
    $endgroup$
    – Samuel
    Jan 29 '15 at 22:05






  • 2




    $begingroup$
    Put $f(x) = e^{g(x)}$ then the equation becomes $g(x) + g(y) = g(x+y)$. The solution to this last equation can be found here (which shows that the only continuous solution is $g(x)=kx$)
    $endgroup$
    – Winther
    Jan 29 '15 at 22:09




















  • $begingroup$
    Which set is $A$?
    $endgroup$
    – ajotatxe
    Jan 29 '15 at 21:58










  • $begingroup$
    For my case, it's $U(1) = lbrace e^{ivarphi} mid varphi in left[ 0, 2pi right] rbrace$ which can be represented by $ varphi in left[ 0, 2pi right]$ so I guess that A is $left[ 0, 2pi right]$
    $endgroup$
    – Samuel
    Jan 29 '15 at 22:02








  • 1




    $begingroup$
    @Samuel I would disagree with you on that, $U(1)$ is the circle, which is certainly different from $[0,2pi]$. In particular, $0$ and $2pi$ have the same image under $varphimapsto e^{ivarphi}$.
    $endgroup$
    – Hayden
    Jan 29 '15 at 22:04










  • $begingroup$
    So $left[ 0, 2pi right[$ ?
    $endgroup$
    – Samuel
    Jan 29 '15 at 22:05






  • 2




    $begingroup$
    Put $f(x) = e^{g(x)}$ then the equation becomes $g(x) + g(y) = g(x+y)$. The solution to this last equation can be found here (which shows that the only continuous solution is $g(x)=kx$)
    $endgroup$
    – Winther
    Jan 29 '15 at 22:09


















$begingroup$
Which set is $A$?
$endgroup$
– ajotatxe
Jan 29 '15 at 21:58




$begingroup$
Which set is $A$?
$endgroup$
– ajotatxe
Jan 29 '15 at 21:58












$begingroup$
For my case, it's $U(1) = lbrace e^{ivarphi} mid varphi in left[ 0, 2pi right] rbrace$ which can be represented by $ varphi in left[ 0, 2pi right]$ so I guess that A is $left[ 0, 2pi right]$
$endgroup$
– Samuel
Jan 29 '15 at 22:02






$begingroup$
For my case, it's $U(1) = lbrace e^{ivarphi} mid varphi in left[ 0, 2pi right] rbrace$ which can be represented by $ varphi in left[ 0, 2pi right]$ so I guess that A is $left[ 0, 2pi right]$
$endgroup$
– Samuel
Jan 29 '15 at 22:02






1




1




$begingroup$
@Samuel I would disagree with you on that, $U(1)$ is the circle, which is certainly different from $[0,2pi]$. In particular, $0$ and $2pi$ have the same image under $varphimapsto e^{ivarphi}$.
$endgroup$
– Hayden
Jan 29 '15 at 22:04




$begingroup$
@Samuel I would disagree with you on that, $U(1)$ is the circle, which is certainly different from $[0,2pi]$. In particular, $0$ and $2pi$ have the same image under $varphimapsto e^{ivarphi}$.
$endgroup$
– Hayden
Jan 29 '15 at 22:04












$begingroup$
So $left[ 0, 2pi right[$ ?
$endgroup$
– Samuel
Jan 29 '15 at 22:05




$begingroup$
So $left[ 0, 2pi right[$ ?
$endgroup$
– Samuel
Jan 29 '15 at 22:05




2




2




$begingroup$
Put $f(x) = e^{g(x)}$ then the equation becomes $g(x) + g(y) = g(x+y)$. The solution to this last equation can be found here (which shows that the only continuous solution is $g(x)=kx$)
$endgroup$
– Winther
Jan 29 '15 at 22:09






$begingroup$
Put $f(x) = e^{g(x)}$ then the equation becomes $g(x) + g(y) = g(x+y)$. The solution to this last equation can be found here (which shows that the only continuous solution is $g(x)=kx$)
$endgroup$
– Winther
Jan 29 '15 at 22:09












3 Answers
3






active

oldest

votes


















1












$begingroup$

The general solution of the given functional equation depends on conditions imposed to $f$.
We have:



The general continuous non vanishing complex solution of $f(x+y)=f(x)f(y)$ is $exp(ax+bbar x)$, where $bar x$ is the complex conjugate of $x$. Note that this function is not holomorphic.



If we request that $f$ be differentiable then the general solution is $f(x)=exp (ax)$, as proved in the @doetoe answer.





Added:



here I give a similar but a bit simpler proof:



Note that $f(x+y)=f(x)f(y) Rightarrow f(x)=f(x+0)=f(x)f(0) Rightarrow f(0)=1$ ( if $f$ is not the null function). We have also: $f'(x+y)=f'(x)f(y)$ (where $f'$ is the derivative with respect to $x$).
Now, setting $x=0$ and dividing by $f(y)$ yields:
$$
f'(y)=f'(0)f(y) Rightarrow f(y)=kexpleft(f'(0)yright)
$$
and, given $f(0)=1$ we must have $k=1$.





The continuity request can be weakened but, if $f$ is not measurable then, with the the aid of an Hamel basis, we can find more general ''wild'' solutions.



For a proof see: J.Aczél : lectures on functional equations and their applications, pag. 216.






share|cite|improve this answer











$endgroup$





















    3












    $begingroup$

    If we assume differentiability on the outset, there is one approach that I like that is quite general to turn it into a differential equation: consider the map $Bbb R^2toBbb R^2$ defined by



    $$(x,y)mapsto (f(x)f(y), f(x+y)).$$



    The functional equation implies that the image is one dimensional, so the Jacobian is everywhere degenerate. Working it out we see:



    $$f'(x)f(y)f'(x+y) = f'(x+y)f(x)f'(y)$$



    from which you can easily argue that



    $$frac d{dx}log f(x) = frac{f'(x)}{f(x)} = frac{f'(0)}{f(0)} = constant$$



    so that the result follows (in the differentiable case).






    share|cite|improve this answer









    $endgroup$





















      1












      $begingroup$

      Let $y=1$ and $f(1)=e^k$ for some $kinmathbb{C}$, then
      $$
      f(x+1)=e^kf(x)\
      f(x)=e^kf(x-1)\
      f(x)=e^{2k}f(x-2)\
      f(x)=e^{3k}f(x-3)\
      vdots\
      f(x)=e^{(x-1)k}f(1)\
      f(x)=e^{kx}\
      $$






      share|cite|improve this answer











      $endgroup$













      • $begingroup$
        This derivation holds only for integer $x$.
        $endgroup$
        – Winther
        Jan 30 '15 at 8:09












      • $begingroup$
        @Winther. No, because you do not have to assume that $f(1)=e^k$. You can assume that for some $ainmathbb{R},f(a)=e^k$.
        $endgroup$
        – user164524
        Jan 30 '15 at 9:18












      • $begingroup$
        I'm referring to the $f(x-3)cdots f(1)$ step which only works for $x$ integer.
        $endgroup$
        – Winther
        Jan 30 '15 at 9:33












      • $begingroup$
        @Winther. It is just an example. Because $f(x+y)=f(x)f(y)$ is true for all $x,y$, we can let $y$ have some positive value very close to $0$, so we have $yto0^+$ and $f(y)=e^k$.
        $endgroup$
        – user164524
        Jan 30 '15 at 9:50








      • 2




        $begingroup$
        That is not correct. There are other (strange) solutions to this equation. It's only when continuity is assumed that this becomes the only solution. You can read more about that for a similar equation here
        $endgroup$
        – Winther
        Jan 30 '15 at 9:53












      Your Answer





      StackExchange.ifUsing("editor", function () {
      return StackExchange.using("mathjaxEditing", function () {
      StackExchange.MarkdownEditor.creationCallbacks.add(function (editor, postfix) {
      StackExchange.mathjaxEditing.prepareWmdForMathJax(editor, postfix, [["$", "$"], ["\\(","\\)"]]);
      });
      });
      }, "mathjax-editing");

      StackExchange.ready(function() {
      var channelOptions = {
      tags: "".split(" "),
      id: "69"
      };
      initTagRenderer("".split(" "), "".split(" "), channelOptions);

      StackExchange.using("externalEditor", function() {
      // Have to fire editor after snippets, if snippets enabled
      if (StackExchange.settings.snippets.snippetsEnabled) {
      StackExchange.using("snippets", function() {
      createEditor();
      });
      }
      else {
      createEditor();
      }
      });

      function createEditor() {
      StackExchange.prepareEditor({
      heartbeatType: 'answer',
      autoActivateHeartbeat: false,
      convertImagesToLinks: true,
      noModals: true,
      showLowRepImageUploadWarning: true,
      reputationToPostImages: 10,
      bindNavPrevention: true,
      postfix: "",
      imageUploader: {
      brandingHtml: "Powered by u003ca class="icon-imgur-white" href="https://imgur.com/"u003eu003c/au003e",
      contentPolicyHtml: "User contributions licensed under u003ca href="https://creativecommons.org/licenses/by-sa/3.0/"u003ecc by-sa 3.0 with attribution requiredu003c/au003e u003ca href="https://stackoverflow.com/legal/content-policy"u003e(content policy)u003c/au003e",
      allowUrls: true
      },
      noCode: true, onDemand: true,
      discardSelector: ".discard-answer"
      ,immediatelyShowMarkdownHelp:true
      });


      }
      });














      draft saved

      draft discarded


















      StackExchange.ready(
      function () {
      StackExchange.openid.initPostLogin('.new-post-login', 'https%3a%2f%2fmath.stackexchange.com%2fquestions%2f1125644%2fsolving-functional-equation-fxfy-fxy%23new-answer', 'question_page');
      }
      );

      Post as a guest















      Required, but never shown

























      3 Answers
      3






      active

      oldest

      votes








      3 Answers
      3






      active

      oldest

      votes









      active

      oldest

      votes






      active

      oldest

      votes









      1












      $begingroup$

      The general solution of the given functional equation depends on conditions imposed to $f$.
      We have:



      The general continuous non vanishing complex solution of $f(x+y)=f(x)f(y)$ is $exp(ax+bbar x)$, where $bar x$ is the complex conjugate of $x$. Note that this function is not holomorphic.



      If we request that $f$ be differentiable then the general solution is $f(x)=exp (ax)$, as proved in the @doetoe answer.





      Added:



      here I give a similar but a bit simpler proof:



      Note that $f(x+y)=f(x)f(y) Rightarrow f(x)=f(x+0)=f(x)f(0) Rightarrow f(0)=1$ ( if $f$ is not the null function). We have also: $f'(x+y)=f'(x)f(y)$ (where $f'$ is the derivative with respect to $x$).
      Now, setting $x=0$ and dividing by $f(y)$ yields:
      $$
      f'(y)=f'(0)f(y) Rightarrow f(y)=kexpleft(f'(0)yright)
      $$
      and, given $f(0)=1$ we must have $k=1$.





      The continuity request can be weakened but, if $f$ is not measurable then, with the the aid of an Hamel basis, we can find more general ''wild'' solutions.



      For a proof see: J.Aczél : lectures on functional equations and their applications, pag. 216.






      share|cite|improve this answer











      $endgroup$


















        1












        $begingroup$

        The general solution of the given functional equation depends on conditions imposed to $f$.
        We have:



        The general continuous non vanishing complex solution of $f(x+y)=f(x)f(y)$ is $exp(ax+bbar x)$, where $bar x$ is the complex conjugate of $x$. Note that this function is not holomorphic.



        If we request that $f$ be differentiable then the general solution is $f(x)=exp (ax)$, as proved in the @doetoe answer.





        Added:



        here I give a similar but a bit simpler proof:



        Note that $f(x+y)=f(x)f(y) Rightarrow f(x)=f(x+0)=f(x)f(0) Rightarrow f(0)=1$ ( if $f$ is not the null function). We have also: $f'(x+y)=f'(x)f(y)$ (where $f'$ is the derivative with respect to $x$).
        Now, setting $x=0$ and dividing by $f(y)$ yields:
        $$
        f'(y)=f'(0)f(y) Rightarrow f(y)=kexpleft(f'(0)yright)
        $$
        and, given $f(0)=1$ we must have $k=1$.





        The continuity request can be weakened but, if $f$ is not measurable then, with the the aid of an Hamel basis, we can find more general ''wild'' solutions.



        For a proof see: J.Aczél : lectures on functional equations and their applications, pag. 216.






        share|cite|improve this answer











        $endgroup$
















          1












          1








          1





          $begingroup$

          The general solution of the given functional equation depends on conditions imposed to $f$.
          We have:



          The general continuous non vanishing complex solution of $f(x+y)=f(x)f(y)$ is $exp(ax+bbar x)$, where $bar x$ is the complex conjugate of $x$. Note that this function is not holomorphic.



          If we request that $f$ be differentiable then the general solution is $f(x)=exp (ax)$, as proved in the @doetoe answer.





          Added:



          here I give a similar but a bit simpler proof:



          Note that $f(x+y)=f(x)f(y) Rightarrow f(x)=f(x+0)=f(x)f(0) Rightarrow f(0)=1$ ( if $f$ is not the null function). We have also: $f'(x+y)=f'(x)f(y)$ (where $f'$ is the derivative with respect to $x$).
          Now, setting $x=0$ and dividing by $f(y)$ yields:
          $$
          f'(y)=f'(0)f(y) Rightarrow f(y)=kexpleft(f'(0)yright)
          $$
          and, given $f(0)=1$ we must have $k=1$.





          The continuity request can be weakened but, if $f$ is not measurable then, with the the aid of an Hamel basis, we can find more general ''wild'' solutions.



          For a proof see: J.Aczél : lectures on functional equations and their applications, pag. 216.






          share|cite|improve this answer











          $endgroup$



          The general solution of the given functional equation depends on conditions imposed to $f$.
          We have:



          The general continuous non vanishing complex solution of $f(x+y)=f(x)f(y)$ is $exp(ax+bbar x)$, where $bar x$ is the complex conjugate of $x$. Note that this function is not holomorphic.



          If we request that $f$ be differentiable then the general solution is $f(x)=exp (ax)$, as proved in the @doetoe answer.





          Added:



          here I give a similar but a bit simpler proof:



          Note that $f(x+y)=f(x)f(y) Rightarrow f(x)=f(x+0)=f(x)f(0) Rightarrow f(0)=1$ ( if $f$ is not the null function). We have also: $f'(x+y)=f'(x)f(y)$ (where $f'$ is the derivative with respect to $x$).
          Now, setting $x=0$ and dividing by $f(y)$ yields:
          $$
          f'(y)=f'(0)f(y) Rightarrow f(y)=kexpleft(f'(0)yright)
          $$
          and, given $f(0)=1$ we must have $k=1$.





          The continuity request can be weakened but, if $f$ is not measurable then, with the the aid of an Hamel basis, we can find more general ''wild'' solutions.



          For a proof see: J.Aczél : lectures on functional equations and their applications, pag. 216.







          share|cite|improve this answer














          share|cite|improve this answer



          share|cite|improve this answer








          edited Jan 30 '15 at 13:20

























          answered Jan 30 '15 at 12:37









          Emilio NovatiEmilio Novati

          52.2k43574




          52.2k43574























              3












              $begingroup$

              If we assume differentiability on the outset, there is one approach that I like that is quite general to turn it into a differential equation: consider the map $Bbb R^2toBbb R^2$ defined by



              $$(x,y)mapsto (f(x)f(y), f(x+y)).$$



              The functional equation implies that the image is one dimensional, so the Jacobian is everywhere degenerate. Working it out we see:



              $$f'(x)f(y)f'(x+y) = f'(x+y)f(x)f'(y)$$



              from which you can easily argue that



              $$frac d{dx}log f(x) = frac{f'(x)}{f(x)} = frac{f'(0)}{f(0)} = constant$$



              so that the result follows (in the differentiable case).






              share|cite|improve this answer









              $endgroup$


















                3












                $begingroup$

                If we assume differentiability on the outset, there is one approach that I like that is quite general to turn it into a differential equation: consider the map $Bbb R^2toBbb R^2$ defined by



                $$(x,y)mapsto (f(x)f(y), f(x+y)).$$



                The functional equation implies that the image is one dimensional, so the Jacobian is everywhere degenerate. Working it out we see:



                $$f'(x)f(y)f'(x+y) = f'(x+y)f(x)f'(y)$$



                from which you can easily argue that



                $$frac d{dx}log f(x) = frac{f'(x)}{f(x)} = frac{f'(0)}{f(0)} = constant$$



                so that the result follows (in the differentiable case).






                share|cite|improve this answer









                $endgroup$
















                  3












                  3








                  3





                  $begingroup$

                  If we assume differentiability on the outset, there is one approach that I like that is quite general to turn it into a differential equation: consider the map $Bbb R^2toBbb R^2$ defined by



                  $$(x,y)mapsto (f(x)f(y), f(x+y)).$$



                  The functional equation implies that the image is one dimensional, so the Jacobian is everywhere degenerate. Working it out we see:



                  $$f'(x)f(y)f'(x+y) = f'(x+y)f(x)f'(y)$$



                  from which you can easily argue that



                  $$frac d{dx}log f(x) = frac{f'(x)}{f(x)} = frac{f'(0)}{f(0)} = constant$$



                  so that the result follows (in the differentiable case).






                  share|cite|improve this answer









                  $endgroup$



                  If we assume differentiability on the outset, there is one approach that I like that is quite general to turn it into a differential equation: consider the map $Bbb R^2toBbb R^2$ defined by



                  $$(x,y)mapsto (f(x)f(y), f(x+y)).$$



                  The functional equation implies that the image is one dimensional, so the Jacobian is everywhere degenerate. Working it out we see:



                  $$f'(x)f(y)f'(x+y) = f'(x+y)f(x)f'(y)$$



                  from which you can easily argue that



                  $$frac d{dx}log f(x) = frac{f'(x)}{f(x)} = frac{f'(0)}{f(0)} = constant$$



                  so that the result follows (in the differentiable case).







                  share|cite|improve this answer












                  share|cite|improve this answer



                  share|cite|improve this answer










                  answered Jan 29 '15 at 22:53









                  doetoedoetoe

                  1,348811




                  1,348811























                      1












                      $begingroup$

                      Let $y=1$ and $f(1)=e^k$ for some $kinmathbb{C}$, then
                      $$
                      f(x+1)=e^kf(x)\
                      f(x)=e^kf(x-1)\
                      f(x)=e^{2k}f(x-2)\
                      f(x)=e^{3k}f(x-3)\
                      vdots\
                      f(x)=e^{(x-1)k}f(1)\
                      f(x)=e^{kx}\
                      $$






                      share|cite|improve this answer











                      $endgroup$













                      • $begingroup$
                        This derivation holds only for integer $x$.
                        $endgroup$
                        – Winther
                        Jan 30 '15 at 8:09












                      • $begingroup$
                        @Winther. No, because you do not have to assume that $f(1)=e^k$. You can assume that for some $ainmathbb{R},f(a)=e^k$.
                        $endgroup$
                        – user164524
                        Jan 30 '15 at 9:18












                      • $begingroup$
                        I'm referring to the $f(x-3)cdots f(1)$ step which only works for $x$ integer.
                        $endgroup$
                        – Winther
                        Jan 30 '15 at 9:33












                      • $begingroup$
                        @Winther. It is just an example. Because $f(x+y)=f(x)f(y)$ is true for all $x,y$, we can let $y$ have some positive value very close to $0$, so we have $yto0^+$ and $f(y)=e^k$.
                        $endgroup$
                        – user164524
                        Jan 30 '15 at 9:50








                      • 2




                        $begingroup$
                        That is not correct. There are other (strange) solutions to this equation. It's only when continuity is assumed that this becomes the only solution. You can read more about that for a similar equation here
                        $endgroup$
                        – Winther
                        Jan 30 '15 at 9:53
















                      1












                      $begingroup$

                      Let $y=1$ and $f(1)=e^k$ for some $kinmathbb{C}$, then
                      $$
                      f(x+1)=e^kf(x)\
                      f(x)=e^kf(x-1)\
                      f(x)=e^{2k}f(x-2)\
                      f(x)=e^{3k}f(x-3)\
                      vdots\
                      f(x)=e^{(x-1)k}f(1)\
                      f(x)=e^{kx}\
                      $$






                      share|cite|improve this answer











                      $endgroup$













                      • $begingroup$
                        This derivation holds only for integer $x$.
                        $endgroup$
                        – Winther
                        Jan 30 '15 at 8:09












                      • $begingroup$
                        @Winther. No, because you do not have to assume that $f(1)=e^k$. You can assume that for some $ainmathbb{R},f(a)=e^k$.
                        $endgroup$
                        – user164524
                        Jan 30 '15 at 9:18












                      • $begingroup$
                        I'm referring to the $f(x-3)cdots f(1)$ step which only works for $x$ integer.
                        $endgroup$
                        – Winther
                        Jan 30 '15 at 9:33












                      • $begingroup$
                        @Winther. It is just an example. Because $f(x+y)=f(x)f(y)$ is true for all $x,y$, we can let $y$ have some positive value very close to $0$, so we have $yto0^+$ and $f(y)=e^k$.
                        $endgroup$
                        – user164524
                        Jan 30 '15 at 9:50








                      • 2




                        $begingroup$
                        That is not correct. There are other (strange) solutions to this equation. It's only when continuity is assumed that this becomes the only solution. You can read more about that for a similar equation here
                        $endgroup$
                        – Winther
                        Jan 30 '15 at 9:53














                      1












                      1








                      1





                      $begingroup$

                      Let $y=1$ and $f(1)=e^k$ for some $kinmathbb{C}$, then
                      $$
                      f(x+1)=e^kf(x)\
                      f(x)=e^kf(x-1)\
                      f(x)=e^{2k}f(x-2)\
                      f(x)=e^{3k}f(x-3)\
                      vdots\
                      f(x)=e^{(x-1)k}f(1)\
                      f(x)=e^{kx}\
                      $$






                      share|cite|improve this answer











                      $endgroup$



                      Let $y=1$ and $f(1)=e^k$ for some $kinmathbb{C}$, then
                      $$
                      f(x+1)=e^kf(x)\
                      f(x)=e^kf(x-1)\
                      f(x)=e^{2k}f(x-2)\
                      f(x)=e^{3k}f(x-3)\
                      vdots\
                      f(x)=e^{(x-1)k}f(1)\
                      f(x)=e^{kx}\
                      $$







                      share|cite|improve this answer














                      share|cite|improve this answer



                      share|cite|improve this answer








                      edited Jan 30 '15 at 9:21

























                      answered Jan 29 '15 at 23:00







                      user164524



















                      • $begingroup$
                        This derivation holds only for integer $x$.
                        $endgroup$
                        – Winther
                        Jan 30 '15 at 8:09












                      • $begingroup$
                        @Winther. No, because you do not have to assume that $f(1)=e^k$. You can assume that for some $ainmathbb{R},f(a)=e^k$.
                        $endgroup$
                        – user164524
                        Jan 30 '15 at 9:18












                      • $begingroup$
                        I'm referring to the $f(x-3)cdots f(1)$ step which only works for $x$ integer.
                        $endgroup$
                        – Winther
                        Jan 30 '15 at 9:33












                      • $begingroup$
                        @Winther. It is just an example. Because $f(x+y)=f(x)f(y)$ is true for all $x,y$, we can let $y$ have some positive value very close to $0$, so we have $yto0^+$ and $f(y)=e^k$.
                        $endgroup$
                        – user164524
                        Jan 30 '15 at 9:50








                      • 2




                        $begingroup$
                        That is not correct. There are other (strange) solutions to this equation. It's only when continuity is assumed that this becomes the only solution. You can read more about that for a similar equation here
                        $endgroup$
                        – Winther
                        Jan 30 '15 at 9:53


















                      • $begingroup$
                        This derivation holds only for integer $x$.
                        $endgroup$
                        – Winther
                        Jan 30 '15 at 8:09












                      • $begingroup$
                        @Winther. No, because you do not have to assume that $f(1)=e^k$. You can assume that for some $ainmathbb{R},f(a)=e^k$.
                        $endgroup$
                        – user164524
                        Jan 30 '15 at 9:18












                      • $begingroup$
                        I'm referring to the $f(x-3)cdots f(1)$ step which only works for $x$ integer.
                        $endgroup$
                        – Winther
                        Jan 30 '15 at 9:33












                      • $begingroup$
                        @Winther. It is just an example. Because $f(x+y)=f(x)f(y)$ is true for all $x,y$, we can let $y$ have some positive value very close to $0$, so we have $yto0^+$ and $f(y)=e^k$.
                        $endgroup$
                        – user164524
                        Jan 30 '15 at 9:50








                      • 2




                        $begingroup$
                        That is not correct. There are other (strange) solutions to this equation. It's only when continuity is assumed that this becomes the only solution. You can read more about that for a similar equation here
                        $endgroup$
                        – Winther
                        Jan 30 '15 at 9:53
















                      $begingroup$
                      This derivation holds only for integer $x$.
                      $endgroup$
                      – Winther
                      Jan 30 '15 at 8:09






                      $begingroup$
                      This derivation holds only for integer $x$.
                      $endgroup$
                      – Winther
                      Jan 30 '15 at 8:09














                      $begingroup$
                      @Winther. No, because you do not have to assume that $f(1)=e^k$. You can assume that for some $ainmathbb{R},f(a)=e^k$.
                      $endgroup$
                      – user164524
                      Jan 30 '15 at 9:18






                      $begingroup$
                      @Winther. No, because you do not have to assume that $f(1)=e^k$. You can assume that for some $ainmathbb{R},f(a)=e^k$.
                      $endgroup$
                      – user164524
                      Jan 30 '15 at 9:18














                      $begingroup$
                      I'm referring to the $f(x-3)cdots f(1)$ step which only works for $x$ integer.
                      $endgroup$
                      – Winther
                      Jan 30 '15 at 9:33






                      $begingroup$
                      I'm referring to the $f(x-3)cdots f(1)$ step which only works for $x$ integer.
                      $endgroup$
                      – Winther
                      Jan 30 '15 at 9:33














                      $begingroup$
                      @Winther. It is just an example. Because $f(x+y)=f(x)f(y)$ is true for all $x,y$, we can let $y$ have some positive value very close to $0$, so we have $yto0^+$ and $f(y)=e^k$.
                      $endgroup$
                      – user164524
                      Jan 30 '15 at 9:50






                      $begingroup$
                      @Winther. It is just an example. Because $f(x+y)=f(x)f(y)$ is true for all $x,y$, we can let $y$ have some positive value very close to $0$, so we have $yto0^+$ and $f(y)=e^k$.
                      $endgroup$
                      – user164524
                      Jan 30 '15 at 9:50






                      2




                      2




                      $begingroup$
                      That is not correct. There are other (strange) solutions to this equation. It's only when continuity is assumed that this becomes the only solution. You can read more about that for a similar equation here
                      $endgroup$
                      – Winther
                      Jan 30 '15 at 9:53




                      $begingroup$
                      That is not correct. There are other (strange) solutions to this equation. It's only when continuity is assumed that this becomes the only solution. You can read more about that for a similar equation here
                      $endgroup$
                      – Winther
                      Jan 30 '15 at 9:53


















                      draft saved

                      draft discarded




















































                      Thanks for contributing an answer to Mathematics Stack Exchange!


                      • Please be sure to answer the question. Provide details and share your research!

                      But avoid



                      • Asking for help, clarification, or responding to other answers.

                      • Making statements based on opinion; back them up with references or personal experience.


                      Use MathJax to format equations. MathJax reference.


                      To learn more, see our tips on writing great answers.




                      draft saved


                      draft discarded














                      StackExchange.ready(
                      function () {
                      StackExchange.openid.initPostLogin('.new-post-login', 'https%3a%2f%2fmath.stackexchange.com%2fquestions%2f1125644%2fsolving-functional-equation-fxfy-fxy%23new-answer', 'question_page');
                      }
                      );

                      Post as a guest















                      Required, but never shown





















































                      Required, but never shown














                      Required, but never shown












                      Required, but never shown







                      Required, but never shown

































                      Required, but never shown














                      Required, but never shown












                      Required, but never shown







                      Required, but never shown







                      Popular posts from this blog

                      Can a sorcerer learn a 5th-level spell early by creating spell slots using the Font of Magic feature?

                      ts Property 'filter' does not exist on type '{}'

                      Notepad++ export/extract a list of installed plugins